subject
Mathematics, 03.03.2021 17:40 pls391

Write an equation to represent the following statement.
10 less than j is 35.

ansver
Answers: 3

Another question on Mathematics

question
Mathematics, 21.06.2019 19:30
John checked his watch and said that it is thursday, 7 am. what will the day and time be 2006 hours plzzz i will give you 100 points
Answers: 1
question
Mathematics, 21.06.2019 21:00
These box plots show the number of electoral votes one by democratic and republican presidential candidates for the elections from 1984 through 2012. which statement best compares the spread of the data sets
Answers: 2
question
Mathematics, 21.06.2019 23:30
What is the explicit rule for the sequence in simplified form? −1,−4,−7,−10,−13… an=4−5n an=−4−3n an=2−3n an=−6−5n
Answers: 1
question
Mathematics, 22.06.2019 00:00
What is mean median mode and range?
Answers: 2
You know the right answer?
Write an equation to represent the following statement.
10 less than j is 35....
Questions
question
Mathematics, 18.02.2021 17:40
question
Mathematics, 18.02.2021 17:40
question
History, 18.02.2021 17:40
question
Biology, 18.02.2021 17:40
question
Mathematics, 18.02.2021 17:40
question
Mathematics, 18.02.2021 17:40
Questions on the website: 13722360